Last visit was: 23 Apr 2024, 11:02 It is currently 23 Apr 2024, 11:02

Close
GMAT Club Daily Prep
Thank you for using the timer - this advanced tool can estimate your performance and suggest more practice questions. We have subscribed you to Daily Prep Questions via email.

Customized
for You

we will pick new questions that match your level based on your Timer History

Track
Your Progress

every week, we’ll send you an estimated GMAT score based on your performance

Practice
Pays

we will pick new questions that match your level based on your Timer History
Not interested in getting valuable practice questions and articles delivered to your email? No problem, unsubscribe here.
Close
Request Expert Reply
Confirm Cancel
SORT BY:
Date
Tags:
Difficulty: 705-805 Levelx   Algebrax   Arithmeticx                              
Show Tags
Hide Tags
User avatar
Manager
Manager
Joined: 02 Dec 2012
Posts: 172
Own Kudos [?]: 23843 [1444]
Given Kudos: 23
Send PM
Most Helpful Reply
Math Expert
Joined: 02 Sep 2009
Posts: 92875
Own Kudos [?]: 618557 [721]
Given Kudos: 81561
Send PM
Senior Manager
Senior Manager
Joined: 15 Oct 2015
Posts: 375
Own Kudos [?]: 1551 [521]
Given Kudos: 342
Concentration: Finance, Strategy
GPA: 3.93
WE:Account Management (Education)
Send PM
Tutor
Joined: 16 Oct 2010
Posts: 14816
Own Kudos [?]: 64880 [428]
Given Kudos: 426
Location: Pune, India
Send PM
Re: 0.99999999/1.0001 - 0.99999991/1.0003 = [#permalink]
291
Kudos
136
Bookmarks
Expert Reply
Walkabout wrote:
\(\frac{0.99999999}{1.0001}-\frac{0.99999991}{1.0003}=\)

(A) 10^(-8)
(B) 3*10^(-8)
(C) 3*10^(-4)
(D) 2*10^(-4)
(E) 10^(-4)


Responding to a pm:
To be honest, I can't think of an alternative method. The fractions are really complicated and need to be simplified before proceeding. For simplification, I think you will need to use a^2 - b^2 = (a - b)(a + b)

All I can suggest is that you can try to solve it without the exponents if that seems easier e.g.

\(\frac{0.99999999}{1.0001}-\frac{.99999991}{1.0003}\)

\(\frac{{1 - .00000001}}{{1 + .0001}}-\frac{{1 - .00000009}}{{1 + .0003}}\)

\(\frac{{1^2 - .0001^2}}{{1 + .0001}}-\frac{{1^2 - 0.0003^2}}{{1 + .0003}}\)

\(\frac{{(1 - .0001)(1 + .0001)}}{{(1 + .0001)}}-\frac{{(1 - .0003)(1 + .0003)}}{{(1 + .0003)}}\)

\((1 - .0001) - (1 - .0003)\)

\(.0002 = 2*10^{-4}\)
GMAT Club Legend
GMAT Club Legend
Joined: 12 Sep 2015
Posts: 6821
Own Kudos [?]: 29890 [100]
Given Kudos: 799
Location: Canada
Send PM
0.99999999/1.0001 - 0.99999991/1.0003 = [#permalink]
74
Kudos
25
Bookmarks
Expert Reply
Walkabout wrote:
\(\frac{0.99999999}{1.0001}-\frac{0.99999991}{1.0003}=\)

(A) 10^(-8)
(B) 3*10^(-8)
(C) 3*10^(-4)
(D) 2*10^(-4)
(E) 10^(-4)


Approach #1: Algebra

First recognize that \(0.99999999 = 1 - 0.00000001\) and \(0.99999991 = 1 - 0.00000009\)

Also recognize that \(0.00000001 = (0.0001)^2\) and \(0.00000009 = (0.0003)^2\)

So we get: \(\frac{0.99999999}{1.0001}-\frac{0.99999991}{1.0003}=\frac{1 - 0.00000001}{1.0001}-\frac{1 - 0.00000009}{1.0003}\)

\(=\frac{1^2 - (0.0001)^2}{1.0001}-\frac{1^2 - (0.0003)^2}{1.0003}\)

\(=\frac{(1 + 0.0001)(1 - 0.0001)}{1.0001}-\frac{(1 + 0.0003)(1 - 0.0003)}{1.0003}\) [factored as difference of squares]

\(=\frac{(1.0001)(0.9999)}{1.0001}-\frac{(1.0003)(0.9997)}{1.0003}\) [simplified]

\(=0.9999-0.9997\) [some terms cancelled out]

\(=0.0002\)

\(=2 \times 10^{-4}\)

Answer: D

Approach #2: Combine the fractions and then use some approximation.

To avoid so many decimals, let's first multiply the numerators and denominators in both fractions by 10,000 to get the following equivalent fractions: \(\frac{9999.9999}{10001}-\frac{9999.9991}{10003}\)

Rewrite with common denominators: \(\frac{(10003)(9999.9999)}{(10001)(10003)}-\frac{(10001)(9999.9991)}{(10001)(10003)}\)

Combine to get: \(\frac{(10003)(9999.9999) - (10001)(9999.9991)}{(10001)(10003)}\)

Since 9999.9999 and 9999.9991 are very close to 10,000, we get the following approximation: \(\frac{(10003)(10,000) - (10001)(10,000)}{(10001)(10003)}\)

Likewise, we can approximate denominator as follows: \(\frac{(10003)(10,000) - (10001)(10,000)}{(10,000)(10,000)}\)

Factor the numerator: \(\frac{(10,000)(10,003 - 10,001)}{(10,000)(10,000)}\)

Simplify: \(\frac{10,003 - 10,001}{10,000}\)

Simplify: \(\frac{2}{10,000}\)

Rewrite as: \((2)(\frac{1}{10,000})\)

Which is the same as: \((2)(10^{-4})\)

Answer: D

Cheers,
Brent

Originally posted by BrentGMATPrepNow on 10 Apr 2016, 10:51.
Last edited by BrentGMATPrepNow on 25 Oct 2021, 17:24, edited 2 times in total.
Verbal Forum Moderator
Joined: 10 Oct 2012
Posts: 485
Own Kudos [?]: 3091 [51]
Given Kudos: 141
Send PM
Re: 0.99999999/1.0001 - 0.99999991/1.0003 = [#permalink]
43
Kudos
6
Bookmarks
Walkabout wrote:
\(\frac{0.99999999}{1.0001}-\frac{0.99999991}{1.0003}=\)

(A) 10^(-8)
(B) 3*10^(-8)
(C) 3*10^(-4)
(D) 2*10^(-4)
(E) 10^(-4)


The best solution is already outlined by Bunuel/Karishma. Because this is an Official Problem, I was sure there might be another way to do this.
So i did spend some time and realized that 0.99999999 might be a multiple of 1.0001 because of the non-messy options and found this :

9*1.0001 = 9.0009 ; 99*1.0001 = 99.0099 and as because the problem had 9 eight times, we have 9999*1.0001 = 9999.9999.
Again, looking for a similar pattern, the last digit of 0.99999991 gave a hint that maybe we have to multiply by something ending in 7, as because we have 1.0003 in the denominator. And indeed 9997*1.0003 = 9999.9991. Thus, the problem boiled down to \(9999*10^{-4} - 9997*10^{-4} = 2*10^{-4}\)

D.

Maybe a bit of luck was handy.
avatar
Intern
Intern
Joined: 28 May 2012
Posts: 23
Own Kudos [?]: 112 [47]
Given Kudos: 84
Concentration: Finance, General Management
GMAT 1: 700 Q50 V35
GPA: 3.28
WE:Analyst (Investment Banking)
Send PM
Re: 0.99999999/1.0001 - 0.99999991/1.0003 = [#permalink]
37
Kudos
10
Bookmarks
Here is my alternative solution for this problem (not for all problems):

\(\frac{A}{B} - \frac{C}{D}= \frac{(AD-BC)}{BD}\).

So \(\frac{0.99999999}{1.0001} - \frac{0.99999991}{1.0003}= \frac{(0.99999999*1.0003-0.99999991*1.0001)}{(1.0001*1.0003)}\).

For this case, the ultimate digit of 0.99999999*1.0003-0.99999991*1.0001 is 6
In the denominator, the ultimate digit of 1.0001*1.0003 is 3
Therefore, the ultimate digit of the final result is 2. So it should be 2 * 0.00...01 --> Only D has the last digit of 2.

Alternatively, we can calculate each fraction, \(\frac{0.99999999}{1.0001}\) has last digit of 9, and \(\frac{0.99999991}{1.0003}\) has last digit of 7, so the final last digit is 2 --> D

This is a special problem. For example \(\frac{...6}{4}\) can have a result of ...4 or ...9. Therefore, in this case we have to calculate as Bunuel did.

In general, we can only apply this strategy only if the last digit of divisor is 1, 2 or 3.
GMAT Club Legend
GMAT Club Legend
Joined: 19 Dec 2014
Status:GMAT Assassin/Co-Founder
Affiliations: EMPOWERgmat
Posts: 21846
Own Kudos [?]: 11663 [6]
Given Kudos: 450
Location: United States (CA)
GMAT 1: 800 Q51 V49
GRE 1: Q170 V170
Send PM
Re: 0.99999999/1.0001 - 0.99999991/1.0003 = [#permalink]
4
Kudos
2
Bookmarks
Expert Reply
Hi,

To start, since this question has no variables, we know that we're going to be doing some type of math to get to the answer. Looking at the answer choices, they're all written in the same 'format' - and can be rewritten as a decimal point followed by a bunch of 0s and then a single non-0 digit. When the GMAT gives you an 'ugly looking' fraction to work with, you can often 'rewrite' what you've been given (potentially getting rid of the fraction entirely by reducing it or reformatting it).

The first fraction is .99999999/1.0001.... we can multiply both the numerator and denominator by 10,000... which gives us....

9999.9999/10001

You might recognize a pattern here (there will almost certainly be a string of 9s here) Even if you don't spot the pattern though, with a few division steps, you'll end up with .9999

This is interesting for a couple of reasons. First, it's only 4 decimal places (notice how three of the answers fit that pattern while two of them don't). Second, you should again consider the format of the answer choices... each answer is a bunch of 0s followed by a single non-0 digit. That result won't happen if you're subtracting an 8-digit decimal from a 4-digit decimal. Thus, it's almost certain that the second fraction will ALSO be a 4-digit decimal.

Using the same approach that we used on the first fraction, we can rewrite the second fraction as...

9999.9991/10003

Before you do any math, think about how a '3' divides into a '1'.... what will the last digit in this decimal probably be? Since 7x3 = 21, that last digit will almost certainly be a '7'. With a little work, you can prove it. You'll end up with .9997

Subtracting the two decimals, you'll have .9999 - .9997 = .0002

Notice the '2' as the last decimal point? It won't take much to find that answer.

Final Answer:

GMAT assassins aren't born, they're made,
Rich
General Discussion
User avatar
Director
Director
Joined: 22 Mar 2011
Posts: 520
Own Kudos [?]: 2135 [19]
Given Kudos: 43
WE:Science (Education)
Send PM
Re: 0.99999999/1.0001 - 0.99999991/1.0003 = [#permalink]
9
Kudos
9
Bookmarks
\(\frac{0.99999999}{1.0001}-\frac{0.99999991}{1.0003}=\)

A. 10^-8
B. 3(10^-8)
C. 3(10^-4)
D. 2(10^-4)
E. 10^-4[/quote]

Well, you need a little bit of algebra here...

The question is meant to test the formula \(a^2-b^2=(a+b)(a-b).\)
The clue is in the answers, explicitly the powers of \(10\).
You can write the given expression as follows:

\(\frac{1-10^{-8}}{1+10^{-4}}-\frac{1-9*10^{-8}}{1+3*10^{-4}}=\frac{(1+10^{-4})(1-10^{-4})}{1+10^{-4}}-\frac{(1+3*10^{-4})(1-3*10^{-4})}{1+3*10^{-4}}=1-10^{-4}-(1-3*10^{-4})=2*10^{-4}.\)

Answer D
avatar
Intern
Intern
Joined: 09 Sep 2013
Posts: 13
Own Kudos [?]: 9 [3]
Given Kudos: 7
Send PM
Re: 0.99999999/1.0001 - 0.99999991/1.0003 = [#permalink]
3
Kudos
How did we even know to apply a^2 - b^2 = (a - b)(a + b) to this problem? I understand the math, but if I saw this problem on the test I would have never guessed to apply that method.

Thanks,
C
Tutor
Joined: 16 Oct 2010
Posts: 14816
Own Kudos [?]: 64880 [3]
Given Kudos: 426
Location: Pune, India
Send PM
Re: 0.99999999/1.0001 - 0.99999991/1.0003 = [#permalink]
2
Kudos
1
Bookmarks
Expert Reply
runningguy wrote:
How did we even know to apply a^2 - b^2 = (a - b)(a + b) to this problem? I understand the math, but if I saw this problem on the test I would have never guessed to apply that method.

Thanks,
C


(a^2 - b^2) is the "mathematical" method i.e. a very clean solution that a Math Prof will give you. With enough experience a^2 - b^2 method will come to you. But since most of us are not Math professors, we could get through using brute force. Two alternative approaches have been given by mau5 and lequanftu26. You may want to give them a thorough read.
User avatar
Intern
Intern
Joined: 24 Mar 2013
Posts: 49
Own Kudos [?]: 17 [13]
Given Kudos: 10
Send PM
Re: 0.99999999/1.0001 - 0.99999991/1.0003 = [#permalink]
13
Kudos
I just rounded it up, did some questionable math and got lucky, it would seem.

1/1.0001 - 1/1.0003 = ?
1/1.0001 = 1/1.0003
1.0003(1) = 1.0001(1)
1.0003-1.0001=?
0.0002

Answer = D. 2(10^-4)
Alum
Joined: 19 Mar 2012
Posts: 4341
Own Kudos [?]: 51445 [19]
Given Kudos: 2326
Location: United States (WA)
Concentration: Leadership, General Management
Schools: Ross '20 (M)
GMAT 1: 760 Q50 V42
GMAT 2: 740 Q49 V42 (Online)
GMAT 3: 760 Q50 V42 (Online)
GPA: 3.8
WE:Marketing (Non-Profit and Government)
Send PM
Re: 0.99999999/1.0001 - 0.99999991/1.0003 = [#permalink]
10
Kudos
9
Bookmarks
Expert Reply
I'm going to repost Ron's solution

OR
Just do what a fifth grader would do!




Attachment:
00000370.png
00000370.png [ 81.02 KiB | Viewed 136008 times ]
RC & DI Moderator
Joined: 02 Aug 2009
Status:Math and DI Expert
Posts: 11161
Own Kudos [?]: 31860 [13]
Given Kudos: 290
Send PM
Re: 0.99999999/1.0001 - 0.99999991/1.0003 = [#permalink]
10
Kudos
3
Bookmarks
Expert Reply
Walkabout wrote:
\(\frac{0.99999999}{1.0001}-\frac{0.99999991}{1.0003}=\)

(A) 10^(-8)
(B) 3*10^(-8)
(C) 3*10^(-4)
(D) 2*10^(-4)
(E) 10^(-4)



Hi,

we should be able to take advantage of choices whereever possible...
Ofcourse, I donot think choices here were given to be able to eliminate all except ONE...
But then that is what is possible in this Q with the choices given...


\(\frac{0.99999999}{1.0001}-\frac{0.99999991}{1.0003}=\)..
both the terms individually are \(\frac{ODD}{ODD}\)so each term should come out as ODD and \(ODD - ODD =EVEN\)...
\(\frac{0.99999999*1.0003-1.0001*0.99999991}{1.0003*1.0001}=\) should be \(\frac{EVEN}{ODD}\)..
so our answer should be something with the last digit in DECIMALs as some EVEN number..
Only D has a 2 in its ten-thousandths place..
D

But ofcourse if we had another choice of same type, we would have had to use a^2-b^2 as done by bunuel But if choice permits, GMAT is all about using the opportunities...
Target Test Prep Representative
Joined: 04 Mar 2011
Status:Head GMAT Instructor
Affiliations: Target Test Prep
Posts: 3043
Own Kudos [?]: 6270 [13]
Given Kudos: 1646
Send PM
Re: 0.99999999/1.0001 - 0.99999991/1.0003 = [#permalink]
10
Kudos
3
Bookmarks
Expert Reply
Walkabout wrote:
\(\frac{0.99999999}{1.0001}-\frac{0.99999991}{1.0003}=\)

(A) 10^(-8)
(B) 3*10^(-8)
(C) 3*10^(-4)
(D) 2*10^(-4)
(E) 10^(-4)


When first looking at this problem, we must consider the fact that 0.99999999/1.0001 and 0.99999991/1.0003 are both pretty nasty-looking fractions. However, this is a situation in which we can use the idea of the difference of two squares to our advantage. To make this idea a little clearer, let’s first illustrate the concept with a few easier whole numbers. For instance, let’s say we were asked:

999,999/1,001 – 9,991/103 = ?

We could rewrite this as:

(1,000,000 – 1)/1,001 – (10,000 – 9)/103

(1000 + 1)(1000 – 1)/1,001 – (100 – 3)(100 + 3)/103

(1,001)(999)/1,001 – (97)(103)/103

999 – 97 = 902

Notice how cleanly the denominators canceled out in this case. Even though the given problem has decimals, we can follow the same approach.

0.99999999/1.0001 – 0.99999991/1.0003

[(1 – 0.00000001)/1.0001] – [(1 – 0.00000009)/1.0003]

[(1 – 0.0001)(1 + 0.0001)/1.0001] – [(1 – 0.0003)(1 + 0.0003)]

When converting this using the difference of squares, we must be very careful not to make any mistakes with the number of decimal places in our values. Since 0.00000001
has 8 decimal places, the decimals in the factors of the numerator of the first set of brackets must each have 4 decimal places. Similarly, since 0.00000009 has 8 decimal places, the decimals in the factors of the numerator of the second set of brackets must each have 4 decimal places. Let’s continue to simplify.

[(1 – 0.0001)(1 + 0.0001)/1.0001] – [(1 – 0.0003)(1 + 0.0003)]

[(0.9999)(1.0001)/1.0001] – [(0.9997)(1.0003)/1.0003]

0.9999 – 0.9997

0.0002

Converting this to scientific notation to match the answer choices, we have:

2 x 10^-4

Answer is D

Originally posted by JeffTargetTestPrep on 03 May 2016, 09:00.
Last edited by JeffTargetTestPrep on 03 May 2016, 09:19, edited 2 times in total.
avatar
Intern
Intern
Joined: 09 Jun 2017
Posts: 2
Own Kudos [?]: 12 [8]
Given Kudos: 0
Send PM
Re: 0.99999999/1.0001 - 0.99999991/1.0003 = [#permalink]
6
Kudos
2
Bookmarks
Here's how I solved it, which I think is less painful (but maybe wouldn't generalize as well)

\(\frac{0.99999999}{1.0001}-\frac{0.99999991}{1.0003}\)

Get rid of that distracting decimal:

\(\frac{9999999}{100010000}-\frac{99999991}{100030000}\)

Recognize that both fractions are some very small number from 1, so try to expose that small number by pulling out the 1:

\(\frac{100010000-10001}{100010000}-\frac{100030000-30009}{100030000}\)

Simplify, and marvel at the convenient numbers revealed

\(1-\frac{1}{10000}-1+\frac{3}{10000}\)

Do the arithmetic, done.
Intern
Intern
Joined: 27 Mar 2014
Status:Current Student
Posts: 25
Own Kudos [?]: 56 [5]
Given Kudos: 14
Location: United States (IL)
Concentration: Statistics, Marketing
GPA: 3.44
WE:Consulting (Advertising and PR)
Send PM
Re: 0.99999999/1.0001 - 0.99999991/1.0003 = [#permalink]
4
Kudos
1
Bookmarks
Walkabout wrote:
\(\frac{0.99999999}{1.0001}-\frac{0.99999991}{1.0003}=\)


(A) \(10^{(-8)}\)

(B) \(3*10^{(-8)}\)

(C) \(3*10^{(-4)}\)

(D) \(2*10^{(-4)}\)

(E) \(10^{(-4)}\)


Simplest way to handle this kind of problem is:

.99 can be written as \(\frac{99}{10^2}\) OR \(\frac{(10^2-1)}{10^2}\) and .91 can be written as \(\frac{91}{10^2}\) OR \(\frac{(10^2-9)}{10^2}\)

For two 9's, we wrote \(\frac{(10^2-1)}{10^2}\), for eight 9's, we will take \(\frac{(10^8-1)}{10^8}\)
Same rule applies to 1.0001 and 1.0003

So, the simplified version becomes

=> \(\frac{(10^8-1)*10^4}{(10^4+1)*10^8}\) - \(\frac{(10^8-9)*10^4}{(10^4+1)*10^8}\)

=> \(\frac{{(10^4)^2-1^2}}{10^4*(10^4+1)}\) - \(\frac{{(10^4)^2-3^2}}{10^4*(10^4+3)}\)

=> \(\frac{(10^4+1)*(10^4-1)}{10^4*(10^4+1)}\) - \(\frac{(10^4+3)*(10^4-3)}{10^4*(10^4+3)}\)

=> \(\frac{(10^4-1)}{10^4}\) - \(\frac{(10^4-3)}{10^4}\)

=> \(\frac{1}{10^4}*(10^4-1-10^4+3)\)

=> \(2*10^{-4}\)
Tutor
Joined: 16 Oct 2010
Posts: 14816
Own Kudos [?]: 64880 [1]
Given Kudos: 426
Location: Pune, India
Send PM
Re: 0.99999999/1.0001 - 0.99999991/1.0003 = [#permalink]
1
Kudos
Expert Reply
VeritasPrepKarishma wrote:
Walkabout wrote:
\(\frac{0.99999999}{1.0001}-\frac{0.99999991}{1.0003}=\)

(A) 10^(-8)
(B) 3*10^(-8)
(C) 3*10^(-4)
(D) 2*10^(-4)
(E) 10^(-4)


Responding to a pm:
To be honest, I can't think of an alternative method. The fractions are really complicated and need to be simplified before proceeding. For simplification, I think you will need to use a^2 - b^2 = (a - b)(a + b)

All I can suggest is that you can try to solve it without the exponents if that seems easier e.g.

\(\frac{0.99999999}{1.0001}-\frac{.99999991}{1.0003}\)

\(\frac{{1 - .00000001}}{{1 + .0001}}-\frac{{1 - .00000009}}{{1 + .0003}}\)

\(\frac{{1^2 - .0001^2}}{{1 + .0001}}-\frac{{1^2 - 0.0003^2}}{{1 + .0003}}\)

\(\frac{{(1 - .0001)(1 + .0001)}}{{(1 + .0001)}}-\frac{{(1 - .0003)(1 + .0003)}}{{(1 + .0003)}}\)

\((1 - .0001) - (1 - .0003)\)

\(.0002 = 2*10^{-4}\)


Here is the video solution to this difficult looking problem: https://www.gmatclub.com/forum/veritas-prep-resource-links-no-longer-available-399979.html#-soluti ... olving_211
VP
VP
Joined: 09 Mar 2016
Posts: 1160
Own Kudos [?]: 1017 [0]
Given Kudos: 3851
Send PM
Re: 0.99999999/1.0001 - 0.99999991/1.0003 = [#permalink]
Bunuel wrote:
Walkabout wrote:
\(\frac{0.99999999}{1.0001}-\frac{0.99999991}{1.0003}=\)

(A) 10^(-8)
(B) 3*10^(-8)
(C) 3*10^(-4)
(D) 2*10^(-4)
(E) 10^(-4)


\(\frac{0.99999999}{1.0001}-\frac{0.99999991}{1.0003}=\frac{1-10^{-8}}{1+10^{-4}}-\frac{1-9*10^{-8}}{1+3*10^{-4}}\)

Now apply \(a^2-b^2=(a+b)(a-b)\):

\(\frac{1-10^{-8}}{1+10^{-4}}-\frac{1-9*10^{-8}}{1+3*10^{-4}}=\frac{(1+10^{-4})(1-10^{-4})}{1+10^{-4}}-\frac{(1+3*10^{-4})(1-3*10^{-4})}{1+3*10^{-4}}=(1-10^{-4})-(1-3*10^{-4})=2*10^{-4}\).

Answer: D.


Hello Bunuel :-) - do you have some useful links for better understanding of your solution ... i dont understand based on which rule transform 1.0003 into 1+3*10^-4 , and 1-9*10^-8 ?

Also what made you apply this formula :? \(a^2-b^2=(a+b)(a-b)\) and could you you break down this solution into more detaled steps ?
\(\frac{1-10^{-8}}{1+10^{-4}}-\frac{1-9*10^{-8}}{1+3*10^{-4}}=\frac{(1+10^{-4})(1-10^{-4})}{1+10^{-4}}-\frac{(1+3*10^{-4})(1-3*10^{-4})}{1+3*10^{-4}}=(1-10^{-4})-(1-3*10^{-4})=2*10^{-4}\). and what common denominator you chose after applying \(a^2-b^2=(a+b)(a-b)\)

Thank you :-)
Math Expert
Joined: 02 Sep 2009
Posts: 92875
Own Kudos [?]: 618557 [9]
Given Kudos: 81561
Send PM
Re: 0.99999999/1.0001 - 0.99999991/1.0003 = [#permalink]
9
Bookmarks
Expert Reply
dave13 wrote:
Bunuel wrote:
Walkabout wrote:
\(\frac{0.99999999}{1.0001}-\frac{0.99999991}{1.0003}=\)

(A) 10^(-8)
(B) 3*10^(-8)
(C) 3*10^(-4)
(D) 2*10^(-4)
(E) 10^(-4)


\(\frac{0.99999999}{1.0001}-\frac{0.99999991}{1.0003}=\frac{1-10^{-8}}{1+10^{-4}}-\frac{1-9*10^{-8}}{1+3*10^{-4}}\)

Now apply \(a^2-b^2=(a+b)(a-b)\):

\(\frac{1-10^{-8}}{1+10^{-4}}-\frac{1-9*10^{-8}}{1+3*10^{-4}}=\frac{(1+10^{-4})(1-10^{-4})}{1+10^{-4}}-\frac{(1+3*10^{-4})(1-3*10^{-4})}{1+3*10^{-4}}=(1-10^{-4})-(1-3*10^{-4})=2*10^{-4}\).

Answer: D.


Hello Bunuel :-) - do you have some useful links for better understanding of your solution ... i dont understand based on which rule transform 1.0003 into 1+3*10^-4 , and 1-9*10^-8 ?

Also what made you apply this formula :? \(a^2-b^2=(a+b)(a-b)\) and could you you break down this solution into more detaled steps ?
\(\frac{1-10^{-8}}{1+10^{-4}}-\frac{1-9*10^{-8}}{1+3*10^{-4}}=\frac{(1+10^{-4})(1-10^{-4})}{1+10^{-4}}-\frac{(1+3*10^{-4})(1-3*10^{-4})}{1+3*10^{-4}}=(1-10^{-4})-(1-3*10^{-4})=2*10^{-4}\). and what common denominator you chose after applying \(a^2-b^2=(a+b)(a-b)\)

Thank you :-)


1.
\(1+10^{-4}=1+\frac{1}{1,0000}=1+0.0001=1.0001\)
\(1+3*10^{-4}=1+\frac{3}{1,0000}=1+0.0003=1.0003\)

2. You should apply \(a^2-b^2=(a+b)(a-b)\) because after this you can reduce the denominator.

3.

8. Exponents and Roots of Numbers



Check below for more:
ALL YOU NEED FOR QUANT ! ! !
Ultimate GMAT Quantitative Megathread

Hope it helps.
GMAT Club Bot
Re: 0.99999999/1.0001 - 0.99999991/1.0003 = [#permalink]
 1   2   
Moderators:
Math Expert
92875 posts
Senior Moderator - Masters Forum
3137 posts

Powered by phpBB © phpBB Group | Emoji artwork provided by EmojiOne